240 divide a p^4 - 1, donde p es cualquier primo mayor o igual que 7

240 divide a p^4 - 1, donde p es cualquier primo mayor o igual que 7

de Marco Antonio Perez -
Número de respuestas: 0

Buenas tardes,

Hoy en la consulta se estuvo trabajando el problema de probar que 240 | (p^4 - 1). Intentamos usar el hecho de que p es de la forma 4k \pm 1 seguido de un argumento de inducción, pero no resultó. Las ideas que vimos sirven para resolver esto, pero antes debe simplificarse el problema. 

Primero notemos que lo que se pide demostrar es equivalente a probar que la congruencia p^4 \equiv 1 ({\rm mod} \ 240) se cumple. Por otro lado, como 240 = 3 \cdot 5 \cdot 16 y además 3, 5 y 16 son coprimos dos a dos, demostrar la congruencia p^4 \equiv 1 ({\rm mod} \ 240) es equivalente a demostrar que se cumplen las siguientes tres congruencias de forma simultánea:

(I) p^4 \equiv 1 ({\rm mod} \ 3)
(II) p^4 \equiv 1 ({\rm mod} \ 5), y
(III) p^4 \equiv 1 ({\rm mod} \ 16).

Como p es cualquier primo mayor o igual que 7, tenemos que p es coprimo con 3, 5 y 16. Por el Teorema de Fermat-Euler, tenemos que p^{\varphi(3)} \equiv 1 ({\rm mod} \ 3)p^{\varphi(5)} \equiv 1 ({\rm mod} \ 5), donde \varphi(3) = 2 y \varphi(5) = 4, ya que 3 y 5 son primos. Entonces, p^2 \equiv 1 ({\rm mod} \ 3), y si elevamos al cuadrado esta congruencia se obtiene p^4 \equiv 1 ({\rm mod} \ 3), es decir, se cumple (I). Por otro lado, de p^{\varphi(5)} \equiv 1 ({\rm mod} \ 5) se tiene directamente que p^{4} \equiv 1 ({\rm mod} \ 5), es decir, se cumple (II). 

La congruencia (III) es la más delicada de demostrar. Para empezar, no nos sirve el Teorema de Fermat-Euler pues \varphi(16) = 8, de donde se tendría p^{8} \equiv 1 ({\rm mod} \ 16), pero lo que necesitamos es p^{4} \equiv 1 ({\rm mod} \ 16). Llegados a este punto, una manera de resolver la congruencia es usando el dato de que p = 4k \pm 1 para algún k \in \mathbb{Z}. En efecto, vemos que:

p^4 = (4k \pm 1)^4 = (4k \pm 1)^2 (4k \pm 1)^2 = 16^2 k^4 \pm 16 \cdot 8 k^2 + 16 k^2 \pm 8 \cdot 16 k^3 + 64 k^2 + (8k \pm 8k) + 16 k^2 + 1 \equiv 1 ({\rm mod} \ 16).

Por lo tanto, la congruencia (III) también se satisface. 

Saludos cordiales,
Marco